LSAT and Law School Admissions Forum

Get expert LSAT preparation and law school admissions advice from PowerScore Test Preparation.

 Administrator
PowerScore Staff
  • PowerScore Staff
  • Posts: 8917
  • Joined: Feb 02, 2011
|
#24989
Complete Question Explanation

Main Point—FIB. The correct answer choice is (C)

This stimulus provides information about the interaction between boat engines and killer whales. The frequency of some noise made by boat engines is the same frequency as the killer whales use to communicate with one another. Though the whales do not seem to act differently around the boats, the noise can damage their hearing. The stimulus does not contain a conclusion, and the question stem asks us to fill in the missing information, which here is the conclusion. We know that we are looking for a conclusion because the missing information is preceded by a conclusion indicator, “therefore.” Additionally, the information in the stimulus is merely a set of facts. In order to be considered a true argument, it needs a conclusion.

Once we know we are looking for the conclusion for the argument, we need to prephrase to better prepare to attack the answer choices. The argument suggests that though the whales do not show any active signs of harm around boat engines, there may be some long term damage. Since the noise of the engines can harm the whales’ hearing, we can draw the conclusion that eventually the engines may impact the whales’ ability to hear the sounds their fellow whales make.

Answer choice (A): Main Point questions are part of the “prove” family of question. The answer choice must be completely supported by the information in the stimulus. The stimulus does not provide information about how younger or older whales hear or react to the sounds from the boats. Thus, this answer choice is not supported by the argument.

Answer choice (B): The stimulus specifically states that the whales do not act differently around the boat engines. This answer choice is directly contradicted by information in the stimulus, and cannot be the Main Point.

Answer choice (C): This is the correct answer choice. Since the boat engines have a harmful impact on the whales’ hearing, it would they also may impact their ability to communicate. The answer choice is phrased in terms of possibility, and by using the term “may” it correctly aligns with the stimulus. The stimulus does not support a stronger conclusion, because it only states that the noise can be loud enough to harm the whales’ hearing. Since the stimulus uses the language of possibility, it is appropriate for the answer choice to use it as well.

Answer choice (D): As with answer choice (A), this answer choice is incorrect as it adds information outside beyond what is supported by the facts in the stimulus. The author does not provide information to support that the whales prefer to be in areas with light boat traffic. If the answer choice is not supported by the facts in the argument, it cannot be the Main Point.

Answer choice (E): The stimulus does not discuss difficulty whales may have with finding food..
Even though this answer choice is stated as a probability, the information is too far removed from the facts in the stimulus to be the main point. There are not sufficient facts in the stimulus to lead us to the conclusion.

Get the most out of your LSAT Prep Plus subscription.

Analyze and track your performance with our Testing and Analytics Package.